Search found 221 matches

by krisraam

Sat Aug 08, 2009 8:45 am
Forum: Critical Reasoning
Topic: The foreign minister of Zeria
Replies: 6
Views: 1840
by krisraam

Sat Aug 08, 2009 8:41 am
Forum: Critical Reasoning
Topic: The proper way to plan
Replies: 11
Views: 2547

IMO A.

Whats the OA.

Thanks
Raama

by krisraam

Sat Aug 08, 2009 8:30 am
Forum: Critical Reasoning
Topic: Underlying assumption CR
Replies: 7
Views: 1946

IMO E.

Thanks
Raama

by krisraam

Sat Aug 08, 2009 8:13 am
Forum: Critical Reasoning
Topic: Patrick
Replies: 13
Views: 2567

IMO D.

Thanks,
Raama

by krisraam

Thu Aug 06, 2009 5:13 pm
Forum: Critical Reasoning
Topic: Cr challenege - 5
Replies: 17
Views: 4623
by krisraam

Tue Jul 28, 2009 10:41 am
Forum: Critical Reasoning
Topic: Fianancial benefit
Replies: 7
Views: 1685
by krisraam

Mon Jul 27, 2009 6:56 pm
Forum: Problem Solving
Topic: Cirle and two perpendicular line
Replies: 3
Views: 1127

(1/2 + 1/2^3 +1/2^5....) - 1/2 (1/2 + 1/2^3 + ...)

= 1/2(1/2 + 1/2^3 + ...)
= 1/4 + 1/4( 1/4 + 1/2^4...)
= 1/4 + 1/4( value will bw < 1)

Sum ranges between 1/4 and 1/2

Thanks
Raama

by krisraam

Mon Jul 27, 2009 6:13 pm
Forum: Problem Solving
Topic: GMAT PREP QUESTION w/OA
Replies: 11
Views: 6076

All the choices lead to real numbers. when x= 1==> sqrt(1) - (Sqrt(2) -Sqrt(1)) ==> 2 -Sqrt(2) Irrational number. But real number. Similarly for all other choices. If the number is not a real number it need to be either imaginary number or complex number. Thanks Raama

by krisraam

Mon Jul 27, 2009 5:47 pm
Forum: Problem Solving
Topic: Sq. Root Problem
Replies: 6
Views: 1537

Let the no of boys be x and girls be y

8x + 7y = 2600

x and y need to be an integer.

x = 150 and y = 200.

Thanks
Raama

by krisraam

Mon Jul 27, 2009 5:17 pm
Forum: Problem Solving
Topic: Percentages
Replies: 1
Views: 1239

On a number line the points will be like this

-------- -k --------0----------k----------

For y to have maximum value x needs to be either equal to or less than -k

y is max when x<=-k

y = 2k

We need k value to find the answer for y

B is sufficient.

Thanks
Raama

by krisraam

Thu Jul 23, 2009 6:17 pm
Forum: Data Sufficiency
Topic: Manhattan GMAT 700+ Challenge Problem - Absolute
Replies: 9
Views: 5219

Sqrt(x-3)^2 = |x-3|

|x-3| = 3-x when x-3 < 0 ie x<3

Stmt 2 says x<0

So B is sufficient.

Thanks
Raama

by krisraam

Wed Jul 22, 2009 6:18 pm
Forum: Data Sufficiency
Topic: Square root and absolute value
Replies: 2
Views: 6112
by krisraam

Thu May 14, 2009 8:19 am
Forum: Sentence Correction
Topic: bright question
Replies: 9
Views: 1590
by krisraam

Thu May 14, 2009 8:13 am
Forum: Problem Solving
Topic: Population Problem
Replies: 3
Views: 2335
by krisraam

Thu May 14, 2009 8:08 am
Forum: Sentence Correction
Topic: regulations
Replies: 10
Views: 3049